Show that $MNPQ$ is a square












1












$begingroup$


Let $ ABCD $ a quadrilateral s.t. $AC=BD $ and $m (angle AOD)=30°$ where $O=ACcap BD $.



Let $triangle ABM, triangle DCN, triangle ADN, triangle CBQ $ equilateral triangles with $Int (triangle ABM)cap Int (ABCD)=emptyset$, $Int (triangle DCP)cap Int (ABCD)=emptyset$, $Int (triangle ADN)cap Int (ABCD) neq emptyset$, $Int (triangle CBQ)cap Int (ABCD)neqemptyset$.



Show that $MNPQ$ is a square.



I have no idea how to start.enter image description here










share|cite|improve this question











$endgroup$












  • $begingroup$
    Just drawn a picture and it didn't look like a square to me.
    $endgroup$
    – Quang Hoang
    Jan 10 at 21:02






  • 1




    $begingroup$
    Edit: I think I see the problem here. All equilateral triangles should be drawn in the opposite direction and the statement would be true. I still don't know how to approach though...
    $endgroup$
    – Edward H.
    Jan 10 at 21:23












  • $begingroup$
    Ok so here's an idea assuming that the equilateral triangles are meant to be drawn oppositely: Rotate ΔABC by 60° around B and you'll get ΔMBQ. Do this 4 times and use AC=BD, and you'll find that MNPQ is a rhombus. Moreover if you keep track of all the angles of rotation and use ∠AOD = 30°, you'll find that the angles of the rhombus are all right, and hence it is a square.
    $endgroup$
    – Edward H.
    Jan 10 at 21:49










  • $begingroup$
    As given it's clearly false. $mangle AMB =60$ and $mangle MBA = 60$ and $mangle QBC = 60$ so $Q$ is in the interior of $angle AMB$. As is $N$ so $mangle NMQ < mangle AMB = 60 < 90$. Can't be a square. But if all the intersections are empty so the equi triangles pop out.
    $endgroup$
    – fleablood
    Jan 10 at 22:29
















1












$begingroup$


Let $ ABCD $ a quadrilateral s.t. $AC=BD $ and $m (angle AOD)=30°$ where $O=ACcap BD $.



Let $triangle ABM, triangle DCN, triangle ADN, triangle CBQ $ equilateral triangles with $Int (triangle ABM)cap Int (ABCD)=emptyset$, $Int (triangle DCP)cap Int (ABCD)=emptyset$, $Int (triangle ADN)cap Int (ABCD) neq emptyset$, $Int (triangle CBQ)cap Int (ABCD)neqemptyset$.



Show that $MNPQ$ is a square.



I have no idea how to start.enter image description here










share|cite|improve this question











$endgroup$












  • $begingroup$
    Just drawn a picture and it didn't look like a square to me.
    $endgroup$
    – Quang Hoang
    Jan 10 at 21:02






  • 1




    $begingroup$
    Edit: I think I see the problem here. All equilateral triangles should be drawn in the opposite direction and the statement would be true. I still don't know how to approach though...
    $endgroup$
    – Edward H.
    Jan 10 at 21:23












  • $begingroup$
    Ok so here's an idea assuming that the equilateral triangles are meant to be drawn oppositely: Rotate ΔABC by 60° around B and you'll get ΔMBQ. Do this 4 times and use AC=BD, and you'll find that MNPQ is a rhombus. Moreover if you keep track of all the angles of rotation and use ∠AOD = 30°, you'll find that the angles of the rhombus are all right, and hence it is a square.
    $endgroup$
    – Edward H.
    Jan 10 at 21:49










  • $begingroup$
    As given it's clearly false. $mangle AMB =60$ and $mangle MBA = 60$ and $mangle QBC = 60$ so $Q$ is in the interior of $angle AMB$. As is $N$ so $mangle NMQ < mangle AMB = 60 < 90$. Can't be a square. But if all the intersections are empty so the equi triangles pop out.
    $endgroup$
    – fleablood
    Jan 10 at 22:29














1












1








1


1



$begingroup$


Let $ ABCD $ a quadrilateral s.t. $AC=BD $ and $m (angle AOD)=30°$ where $O=ACcap BD $.



Let $triangle ABM, triangle DCN, triangle ADN, triangle CBQ $ equilateral triangles with $Int (triangle ABM)cap Int (ABCD)=emptyset$, $Int (triangle DCP)cap Int (ABCD)=emptyset$, $Int (triangle ADN)cap Int (ABCD) neq emptyset$, $Int (triangle CBQ)cap Int (ABCD)neqemptyset$.



Show that $MNPQ$ is a square.



I have no idea how to start.enter image description here










share|cite|improve this question











$endgroup$




Let $ ABCD $ a quadrilateral s.t. $AC=BD $ and $m (angle AOD)=30°$ where $O=ACcap BD $.



Let $triangle ABM, triangle DCN, triangle ADN, triangle CBQ $ equilateral triangles with $Int (triangle ABM)cap Int (ABCD)=emptyset$, $Int (triangle DCP)cap Int (ABCD)=emptyset$, $Int (triangle ADN)cap Int (ABCD) neq emptyset$, $Int (triangle CBQ)cap Int (ABCD)neqemptyset$.



Show that $MNPQ$ is a square.



I have no idea how to start.enter image description here







geometry complex-numbers vectors euclidean-geometry geometric-transformation






share|cite|improve this question















share|cite|improve this question













share|cite|improve this question




share|cite|improve this question








edited Jan 15 at 17:30









Maria Mazur

49.9k1361125




49.9k1361125










asked Jan 10 at 20:51









ProblemsolvingProblemsolving

917412




917412












  • $begingroup$
    Just drawn a picture and it didn't look like a square to me.
    $endgroup$
    – Quang Hoang
    Jan 10 at 21:02






  • 1




    $begingroup$
    Edit: I think I see the problem here. All equilateral triangles should be drawn in the opposite direction and the statement would be true. I still don't know how to approach though...
    $endgroup$
    – Edward H.
    Jan 10 at 21:23












  • $begingroup$
    Ok so here's an idea assuming that the equilateral triangles are meant to be drawn oppositely: Rotate ΔABC by 60° around B and you'll get ΔMBQ. Do this 4 times and use AC=BD, and you'll find that MNPQ is a rhombus. Moreover if you keep track of all the angles of rotation and use ∠AOD = 30°, you'll find that the angles of the rhombus are all right, and hence it is a square.
    $endgroup$
    – Edward H.
    Jan 10 at 21:49










  • $begingroup$
    As given it's clearly false. $mangle AMB =60$ and $mangle MBA = 60$ and $mangle QBC = 60$ so $Q$ is in the interior of $angle AMB$. As is $N$ so $mangle NMQ < mangle AMB = 60 < 90$. Can't be a square. But if all the intersections are empty so the equi triangles pop out.
    $endgroup$
    – fleablood
    Jan 10 at 22:29


















  • $begingroup$
    Just drawn a picture and it didn't look like a square to me.
    $endgroup$
    – Quang Hoang
    Jan 10 at 21:02






  • 1




    $begingroup$
    Edit: I think I see the problem here. All equilateral triangles should be drawn in the opposite direction and the statement would be true. I still don't know how to approach though...
    $endgroup$
    – Edward H.
    Jan 10 at 21:23












  • $begingroup$
    Ok so here's an idea assuming that the equilateral triangles are meant to be drawn oppositely: Rotate ΔABC by 60° around B and you'll get ΔMBQ. Do this 4 times and use AC=BD, and you'll find that MNPQ is a rhombus. Moreover if you keep track of all the angles of rotation and use ∠AOD = 30°, you'll find that the angles of the rhombus are all right, and hence it is a square.
    $endgroup$
    – Edward H.
    Jan 10 at 21:49










  • $begingroup$
    As given it's clearly false. $mangle AMB =60$ and $mangle MBA = 60$ and $mangle QBC = 60$ so $Q$ is in the interior of $angle AMB$. As is $N$ so $mangle NMQ < mangle AMB = 60 < 90$. Can't be a square. But if all the intersections are empty so the equi triangles pop out.
    $endgroup$
    – fleablood
    Jan 10 at 22:29
















$begingroup$
Just drawn a picture and it didn't look like a square to me.
$endgroup$
– Quang Hoang
Jan 10 at 21:02




$begingroup$
Just drawn a picture and it didn't look like a square to me.
$endgroup$
– Quang Hoang
Jan 10 at 21:02




1




1




$begingroup$
Edit: I think I see the problem here. All equilateral triangles should be drawn in the opposite direction and the statement would be true. I still don't know how to approach though...
$endgroup$
– Edward H.
Jan 10 at 21:23






$begingroup$
Edit: I think I see the problem here. All equilateral triangles should be drawn in the opposite direction and the statement would be true. I still don't know how to approach though...
$endgroup$
– Edward H.
Jan 10 at 21:23














$begingroup$
Ok so here's an idea assuming that the equilateral triangles are meant to be drawn oppositely: Rotate ΔABC by 60° around B and you'll get ΔMBQ. Do this 4 times and use AC=BD, and you'll find that MNPQ is a rhombus. Moreover if you keep track of all the angles of rotation and use ∠AOD = 30°, you'll find that the angles of the rhombus are all right, and hence it is a square.
$endgroup$
– Edward H.
Jan 10 at 21:49




$begingroup$
Ok so here's an idea assuming that the equilateral triangles are meant to be drawn oppositely: Rotate ΔABC by 60° around B and you'll get ΔMBQ. Do this 4 times and use AC=BD, and you'll find that MNPQ is a rhombus. Moreover if you keep track of all the angles of rotation and use ∠AOD = 30°, you'll find that the angles of the rhombus are all right, and hence it is a square.
$endgroup$
– Edward H.
Jan 10 at 21:49












$begingroup$
As given it's clearly false. $mangle AMB =60$ and $mangle MBA = 60$ and $mangle QBC = 60$ so $Q$ is in the interior of $angle AMB$. As is $N$ so $mangle NMQ < mangle AMB = 60 < 90$. Can't be a square. But if all the intersections are empty so the equi triangles pop out.
$endgroup$
– fleablood
Jan 10 at 22:29




$begingroup$
As given it's clearly false. $mangle AMB =60$ and $mangle MBA = 60$ and $mangle QBC = 60$ so $Q$ is in the interior of $angle AMB$. As is $N$ so $mangle NMQ < mangle AMB = 60 < 90$. Can't be a square. But if all the intersections are empty so the equi triangles pop out.
$endgroup$
– fleablood
Jan 10 at 22:29










3 Answers
3






active

oldest

votes


















2












$begingroup$

Let $R^{alpha}_O$ be a rotation in the plain by an angle $alpha$ around a point $O$.



Easy to see that to rotate a vector by an angle $alpha$ it's the same to rotate this vector around his tail.



Now, by using the beautiful Daniel Mathias's picture we obtain:
$$R^{90^{circ}}left(vec{NM}right)=R^{30^{circ}}left(R^{60^{circ}}left(vec{NA}+vec{AM}right)right)=R^{30^{circ}}left(vec{DA}+vec{AB}right)=$$
$$=R^{30^{circ}}left(vec{DB}right)=R^{60^{circ}}left(vec{AC}right)=R^{60^{circ}}left(vec{AB}+vec{BC}right)=vec{MB}+vec{BQ}=vec{MQ},$$
which says $NMperp MQ$ and $NM=MQ.$



Also,
$$R^{90^{circ}}left(vec{QP}right)=R^{30^{circ}}left(R^{60^{circ}}left(vec{QC}+vec{CP}right)right)=R^{30^{circ}}left(vec{BC}+vec{CD}right)=$$
$$=R^{30^{circ}}left(vec{BD}right)=R^{60^{circ}}left(vec{CA}right)=R^{60^{circ}}left(vec{CB}+vec{BA}right)=vec{QB}+vec{BM}=vec{QM},$$
which says $QMperp PQ$ and $QM=PQ.$



Can you end it now?






share|cite|improve this answer









$endgroup$













  • $begingroup$
    There is a synthetic proof? I try to prove it with congruences of triangles.
    $endgroup$
    – Problemsolving
    Jan 11 at 5:44






  • 1




    $begingroup$
    I like your solution +1, do you like mine?
    $endgroup$
    – Maria Mazur
    Jan 14 at 20:42










  • $begingroup$
    Do you have some more problems like this solved vith rotation?
    $endgroup$
    – Maria Mazur
    Jan 19 at 11:14












  • $begingroup$
    @greedoid Yes, of course. See here: math.stackexchange.com/questions/3079152
    $endgroup$
    – Michael Rozenberg
    Jan 19 at 11:17










  • $begingroup$
    Yes I saw that one, it is nice. That is all you have? Do you have some pdf file with problems like this?
    $endgroup$
    – Maria Mazur
    Jan 19 at 11:19



















1












$begingroup$

I believe this is the desired result. Note that the intersection/non-intersection is opposite from the description.



enter image description here






share|cite|improve this answer









$endgroup$





















    1












    $begingroup$

    Knowing complex numbers, this one is easy to solve. We need just this lemma:



    Lemma: If $|vec{XY}| =|vec{ZT}|$ and $angle (vec{XY},vec{ZT}) = alpha$ then $$ZT = varepsiloncdot XY$$
    where $varepsilon = cos alpha + i sin alpha$





    So it is enought to prove $MN = icdot MQ;;; (*)$. We have
    $$DB=varepsilon AC ;;;;;;;;{rm where};;;;varepsilon = cos {pi over 6} + i sin {pi over 6} $$
    and if $delta = cos {pi over 3} + i sin {pi over 3} $ then $$MA = delta MB implies M = {A-delta Bover 1-delta}$$
    $$QC = delta QB implies Q = {C-delta Bover 1-delta}$$
    $$NA = delta ND implies N = {A-delta Dover 1-delta}$$



    thus $$MN ={A-delta Dover 1-delta}-{A-delta Bover 1-delta} = {deltaover 1-delta}DB$$ $$ = {deltaover 1-delta}varepsilon cdot AC=i{ACover 1-delta} = iMQ$$



    and we are done.






    share|cite|improve this answer











    $endgroup$














      Your Answer





      StackExchange.ifUsing("editor", function () {
      return StackExchange.using("mathjaxEditing", function () {
      StackExchange.MarkdownEditor.creationCallbacks.add(function (editor, postfix) {
      StackExchange.mathjaxEditing.prepareWmdForMathJax(editor, postfix, [["$", "$"], ["\\(","\\)"]]);
      });
      });
      }, "mathjax-editing");

      StackExchange.ready(function() {
      var channelOptions = {
      tags: "".split(" "),
      id: "69"
      };
      initTagRenderer("".split(" "), "".split(" "), channelOptions);

      StackExchange.using("externalEditor", function() {
      // Have to fire editor after snippets, if snippets enabled
      if (StackExchange.settings.snippets.snippetsEnabled) {
      StackExchange.using("snippets", function() {
      createEditor();
      });
      }
      else {
      createEditor();
      }
      });

      function createEditor() {
      StackExchange.prepareEditor({
      heartbeatType: 'answer',
      autoActivateHeartbeat: false,
      convertImagesToLinks: true,
      noModals: true,
      showLowRepImageUploadWarning: true,
      reputationToPostImages: 10,
      bindNavPrevention: true,
      postfix: "",
      imageUploader: {
      brandingHtml: "Powered by u003ca class="icon-imgur-white" href="https://imgur.com/"u003eu003c/au003e",
      contentPolicyHtml: "User contributions licensed under u003ca href="https://creativecommons.org/licenses/by-sa/3.0/"u003ecc by-sa 3.0 with attribution requiredu003c/au003e u003ca href="https://stackoverflow.com/legal/content-policy"u003e(content policy)u003c/au003e",
      allowUrls: true
      },
      noCode: true, onDemand: true,
      discardSelector: ".discard-answer"
      ,immediatelyShowMarkdownHelp:true
      });


      }
      });














      draft saved

      draft discarded


















      StackExchange.ready(
      function () {
      StackExchange.openid.initPostLogin('.new-post-login', 'https%3a%2f%2fmath.stackexchange.com%2fquestions%2f3069153%2fshow-that-mnpq-is-a-square%23new-answer', 'question_page');
      }
      );

      Post as a guest















      Required, but never shown

























      3 Answers
      3






      active

      oldest

      votes








      3 Answers
      3






      active

      oldest

      votes









      active

      oldest

      votes






      active

      oldest

      votes









      2












      $begingroup$

      Let $R^{alpha}_O$ be a rotation in the plain by an angle $alpha$ around a point $O$.



      Easy to see that to rotate a vector by an angle $alpha$ it's the same to rotate this vector around his tail.



      Now, by using the beautiful Daniel Mathias's picture we obtain:
      $$R^{90^{circ}}left(vec{NM}right)=R^{30^{circ}}left(R^{60^{circ}}left(vec{NA}+vec{AM}right)right)=R^{30^{circ}}left(vec{DA}+vec{AB}right)=$$
      $$=R^{30^{circ}}left(vec{DB}right)=R^{60^{circ}}left(vec{AC}right)=R^{60^{circ}}left(vec{AB}+vec{BC}right)=vec{MB}+vec{BQ}=vec{MQ},$$
      which says $NMperp MQ$ and $NM=MQ.$



      Also,
      $$R^{90^{circ}}left(vec{QP}right)=R^{30^{circ}}left(R^{60^{circ}}left(vec{QC}+vec{CP}right)right)=R^{30^{circ}}left(vec{BC}+vec{CD}right)=$$
      $$=R^{30^{circ}}left(vec{BD}right)=R^{60^{circ}}left(vec{CA}right)=R^{60^{circ}}left(vec{CB}+vec{BA}right)=vec{QB}+vec{BM}=vec{QM},$$
      which says $QMperp PQ$ and $QM=PQ.$



      Can you end it now?






      share|cite|improve this answer









      $endgroup$













      • $begingroup$
        There is a synthetic proof? I try to prove it with congruences of triangles.
        $endgroup$
        – Problemsolving
        Jan 11 at 5:44






      • 1




        $begingroup$
        I like your solution +1, do you like mine?
        $endgroup$
        – Maria Mazur
        Jan 14 at 20:42










      • $begingroup$
        Do you have some more problems like this solved vith rotation?
        $endgroup$
        – Maria Mazur
        Jan 19 at 11:14












      • $begingroup$
        @greedoid Yes, of course. See here: math.stackexchange.com/questions/3079152
        $endgroup$
        – Michael Rozenberg
        Jan 19 at 11:17










      • $begingroup$
        Yes I saw that one, it is nice. That is all you have? Do you have some pdf file with problems like this?
        $endgroup$
        – Maria Mazur
        Jan 19 at 11:19
















      2












      $begingroup$

      Let $R^{alpha}_O$ be a rotation in the plain by an angle $alpha$ around a point $O$.



      Easy to see that to rotate a vector by an angle $alpha$ it's the same to rotate this vector around his tail.



      Now, by using the beautiful Daniel Mathias's picture we obtain:
      $$R^{90^{circ}}left(vec{NM}right)=R^{30^{circ}}left(R^{60^{circ}}left(vec{NA}+vec{AM}right)right)=R^{30^{circ}}left(vec{DA}+vec{AB}right)=$$
      $$=R^{30^{circ}}left(vec{DB}right)=R^{60^{circ}}left(vec{AC}right)=R^{60^{circ}}left(vec{AB}+vec{BC}right)=vec{MB}+vec{BQ}=vec{MQ},$$
      which says $NMperp MQ$ and $NM=MQ.$



      Also,
      $$R^{90^{circ}}left(vec{QP}right)=R^{30^{circ}}left(R^{60^{circ}}left(vec{QC}+vec{CP}right)right)=R^{30^{circ}}left(vec{BC}+vec{CD}right)=$$
      $$=R^{30^{circ}}left(vec{BD}right)=R^{60^{circ}}left(vec{CA}right)=R^{60^{circ}}left(vec{CB}+vec{BA}right)=vec{QB}+vec{BM}=vec{QM},$$
      which says $QMperp PQ$ and $QM=PQ.$



      Can you end it now?






      share|cite|improve this answer









      $endgroup$













      • $begingroup$
        There is a synthetic proof? I try to prove it with congruences of triangles.
        $endgroup$
        – Problemsolving
        Jan 11 at 5:44






      • 1




        $begingroup$
        I like your solution +1, do you like mine?
        $endgroup$
        – Maria Mazur
        Jan 14 at 20:42










      • $begingroup$
        Do you have some more problems like this solved vith rotation?
        $endgroup$
        – Maria Mazur
        Jan 19 at 11:14












      • $begingroup$
        @greedoid Yes, of course. See here: math.stackexchange.com/questions/3079152
        $endgroup$
        – Michael Rozenberg
        Jan 19 at 11:17










      • $begingroup$
        Yes I saw that one, it is nice. That is all you have? Do you have some pdf file with problems like this?
        $endgroup$
        – Maria Mazur
        Jan 19 at 11:19














      2












      2








      2





      $begingroup$

      Let $R^{alpha}_O$ be a rotation in the plain by an angle $alpha$ around a point $O$.



      Easy to see that to rotate a vector by an angle $alpha$ it's the same to rotate this vector around his tail.



      Now, by using the beautiful Daniel Mathias's picture we obtain:
      $$R^{90^{circ}}left(vec{NM}right)=R^{30^{circ}}left(R^{60^{circ}}left(vec{NA}+vec{AM}right)right)=R^{30^{circ}}left(vec{DA}+vec{AB}right)=$$
      $$=R^{30^{circ}}left(vec{DB}right)=R^{60^{circ}}left(vec{AC}right)=R^{60^{circ}}left(vec{AB}+vec{BC}right)=vec{MB}+vec{BQ}=vec{MQ},$$
      which says $NMperp MQ$ and $NM=MQ.$



      Also,
      $$R^{90^{circ}}left(vec{QP}right)=R^{30^{circ}}left(R^{60^{circ}}left(vec{QC}+vec{CP}right)right)=R^{30^{circ}}left(vec{BC}+vec{CD}right)=$$
      $$=R^{30^{circ}}left(vec{BD}right)=R^{60^{circ}}left(vec{CA}right)=R^{60^{circ}}left(vec{CB}+vec{BA}right)=vec{QB}+vec{BM}=vec{QM},$$
      which says $QMperp PQ$ and $QM=PQ.$



      Can you end it now?






      share|cite|improve this answer









      $endgroup$



      Let $R^{alpha}_O$ be a rotation in the plain by an angle $alpha$ around a point $O$.



      Easy to see that to rotate a vector by an angle $alpha$ it's the same to rotate this vector around his tail.



      Now, by using the beautiful Daniel Mathias's picture we obtain:
      $$R^{90^{circ}}left(vec{NM}right)=R^{30^{circ}}left(R^{60^{circ}}left(vec{NA}+vec{AM}right)right)=R^{30^{circ}}left(vec{DA}+vec{AB}right)=$$
      $$=R^{30^{circ}}left(vec{DB}right)=R^{60^{circ}}left(vec{AC}right)=R^{60^{circ}}left(vec{AB}+vec{BC}right)=vec{MB}+vec{BQ}=vec{MQ},$$
      which says $NMperp MQ$ and $NM=MQ.$



      Also,
      $$R^{90^{circ}}left(vec{QP}right)=R^{30^{circ}}left(R^{60^{circ}}left(vec{QC}+vec{CP}right)right)=R^{30^{circ}}left(vec{BC}+vec{CD}right)=$$
      $$=R^{30^{circ}}left(vec{BD}right)=R^{60^{circ}}left(vec{CA}right)=R^{60^{circ}}left(vec{CB}+vec{BA}right)=vec{QB}+vec{BM}=vec{QM},$$
      which says $QMperp PQ$ and $QM=PQ.$



      Can you end it now?







      share|cite|improve this answer












      share|cite|improve this answer



      share|cite|improve this answer










      answered Jan 10 at 23:15









      Michael RozenbergMichael Rozenberg

      110k1896201




      110k1896201












      • $begingroup$
        There is a synthetic proof? I try to prove it with congruences of triangles.
        $endgroup$
        – Problemsolving
        Jan 11 at 5:44






      • 1




        $begingroup$
        I like your solution +1, do you like mine?
        $endgroup$
        – Maria Mazur
        Jan 14 at 20:42










      • $begingroup$
        Do you have some more problems like this solved vith rotation?
        $endgroup$
        – Maria Mazur
        Jan 19 at 11:14












      • $begingroup$
        @greedoid Yes, of course. See here: math.stackexchange.com/questions/3079152
        $endgroup$
        – Michael Rozenberg
        Jan 19 at 11:17










      • $begingroup$
        Yes I saw that one, it is nice. That is all you have? Do you have some pdf file with problems like this?
        $endgroup$
        – Maria Mazur
        Jan 19 at 11:19


















      • $begingroup$
        There is a synthetic proof? I try to prove it with congruences of triangles.
        $endgroup$
        – Problemsolving
        Jan 11 at 5:44






      • 1




        $begingroup$
        I like your solution +1, do you like mine?
        $endgroup$
        – Maria Mazur
        Jan 14 at 20:42










      • $begingroup$
        Do you have some more problems like this solved vith rotation?
        $endgroup$
        – Maria Mazur
        Jan 19 at 11:14












      • $begingroup$
        @greedoid Yes, of course. See here: math.stackexchange.com/questions/3079152
        $endgroup$
        – Michael Rozenberg
        Jan 19 at 11:17










      • $begingroup$
        Yes I saw that one, it is nice. That is all you have? Do you have some pdf file with problems like this?
        $endgroup$
        – Maria Mazur
        Jan 19 at 11:19
















      $begingroup$
      There is a synthetic proof? I try to prove it with congruences of triangles.
      $endgroup$
      – Problemsolving
      Jan 11 at 5:44




      $begingroup$
      There is a synthetic proof? I try to prove it with congruences of triangles.
      $endgroup$
      – Problemsolving
      Jan 11 at 5:44




      1




      1




      $begingroup$
      I like your solution +1, do you like mine?
      $endgroup$
      – Maria Mazur
      Jan 14 at 20:42




      $begingroup$
      I like your solution +1, do you like mine?
      $endgroup$
      – Maria Mazur
      Jan 14 at 20:42












      $begingroup$
      Do you have some more problems like this solved vith rotation?
      $endgroup$
      – Maria Mazur
      Jan 19 at 11:14






      $begingroup$
      Do you have some more problems like this solved vith rotation?
      $endgroup$
      – Maria Mazur
      Jan 19 at 11:14














      $begingroup$
      @greedoid Yes, of course. See here: math.stackexchange.com/questions/3079152
      $endgroup$
      – Michael Rozenberg
      Jan 19 at 11:17




      $begingroup$
      @greedoid Yes, of course. See here: math.stackexchange.com/questions/3079152
      $endgroup$
      – Michael Rozenberg
      Jan 19 at 11:17












      $begingroup$
      Yes I saw that one, it is nice. That is all you have? Do you have some pdf file with problems like this?
      $endgroup$
      – Maria Mazur
      Jan 19 at 11:19




      $begingroup$
      Yes I saw that one, it is nice. That is all you have? Do you have some pdf file with problems like this?
      $endgroup$
      – Maria Mazur
      Jan 19 at 11:19











      1












      $begingroup$

      I believe this is the desired result. Note that the intersection/non-intersection is opposite from the description.



      enter image description here






      share|cite|improve this answer









      $endgroup$


















        1












        $begingroup$

        I believe this is the desired result. Note that the intersection/non-intersection is opposite from the description.



        enter image description here






        share|cite|improve this answer









        $endgroup$
















          1












          1








          1





          $begingroup$

          I believe this is the desired result. Note that the intersection/non-intersection is opposite from the description.



          enter image description here






          share|cite|improve this answer









          $endgroup$



          I believe this is the desired result. Note that the intersection/non-intersection is opposite from the description.



          enter image description here







          share|cite|improve this answer












          share|cite|improve this answer



          share|cite|improve this answer










          answered Jan 10 at 22:08









          Daniel MathiasDaniel Mathias

          1,40518




          1,40518























              1












              $begingroup$

              Knowing complex numbers, this one is easy to solve. We need just this lemma:



              Lemma: If $|vec{XY}| =|vec{ZT}|$ and $angle (vec{XY},vec{ZT}) = alpha$ then $$ZT = varepsiloncdot XY$$
              where $varepsilon = cos alpha + i sin alpha$





              So it is enought to prove $MN = icdot MQ;;; (*)$. We have
              $$DB=varepsilon AC ;;;;;;;;{rm where};;;;varepsilon = cos {pi over 6} + i sin {pi over 6} $$
              and if $delta = cos {pi over 3} + i sin {pi over 3} $ then $$MA = delta MB implies M = {A-delta Bover 1-delta}$$
              $$QC = delta QB implies Q = {C-delta Bover 1-delta}$$
              $$NA = delta ND implies N = {A-delta Dover 1-delta}$$



              thus $$MN ={A-delta Dover 1-delta}-{A-delta Bover 1-delta} = {deltaover 1-delta}DB$$ $$ = {deltaover 1-delta}varepsilon cdot AC=i{ACover 1-delta} = iMQ$$



              and we are done.






              share|cite|improve this answer











              $endgroup$


















                1












                $begingroup$

                Knowing complex numbers, this one is easy to solve. We need just this lemma:



                Lemma: If $|vec{XY}| =|vec{ZT}|$ and $angle (vec{XY},vec{ZT}) = alpha$ then $$ZT = varepsiloncdot XY$$
                where $varepsilon = cos alpha + i sin alpha$





                So it is enought to prove $MN = icdot MQ;;; (*)$. We have
                $$DB=varepsilon AC ;;;;;;;;{rm where};;;;varepsilon = cos {pi over 6} + i sin {pi over 6} $$
                and if $delta = cos {pi over 3} + i sin {pi over 3} $ then $$MA = delta MB implies M = {A-delta Bover 1-delta}$$
                $$QC = delta QB implies Q = {C-delta Bover 1-delta}$$
                $$NA = delta ND implies N = {A-delta Dover 1-delta}$$



                thus $$MN ={A-delta Dover 1-delta}-{A-delta Bover 1-delta} = {deltaover 1-delta}DB$$ $$ = {deltaover 1-delta}varepsilon cdot AC=i{ACover 1-delta} = iMQ$$



                and we are done.






                share|cite|improve this answer











                $endgroup$
















                  1












                  1








                  1





                  $begingroup$

                  Knowing complex numbers, this one is easy to solve. We need just this lemma:



                  Lemma: If $|vec{XY}| =|vec{ZT}|$ and $angle (vec{XY},vec{ZT}) = alpha$ then $$ZT = varepsiloncdot XY$$
                  where $varepsilon = cos alpha + i sin alpha$





                  So it is enought to prove $MN = icdot MQ;;; (*)$. We have
                  $$DB=varepsilon AC ;;;;;;;;{rm where};;;;varepsilon = cos {pi over 6} + i sin {pi over 6} $$
                  and if $delta = cos {pi over 3} + i sin {pi over 3} $ then $$MA = delta MB implies M = {A-delta Bover 1-delta}$$
                  $$QC = delta QB implies Q = {C-delta Bover 1-delta}$$
                  $$NA = delta ND implies N = {A-delta Dover 1-delta}$$



                  thus $$MN ={A-delta Dover 1-delta}-{A-delta Bover 1-delta} = {deltaover 1-delta}DB$$ $$ = {deltaover 1-delta}varepsilon cdot AC=i{ACover 1-delta} = iMQ$$



                  and we are done.






                  share|cite|improve this answer











                  $endgroup$



                  Knowing complex numbers, this one is easy to solve. We need just this lemma:



                  Lemma: If $|vec{XY}| =|vec{ZT}|$ and $angle (vec{XY},vec{ZT}) = alpha$ then $$ZT = varepsiloncdot XY$$
                  where $varepsilon = cos alpha + i sin alpha$





                  So it is enought to prove $MN = icdot MQ;;; (*)$. We have
                  $$DB=varepsilon AC ;;;;;;;;{rm where};;;;varepsilon = cos {pi over 6} + i sin {pi over 6} $$
                  and if $delta = cos {pi over 3} + i sin {pi over 3} $ then $$MA = delta MB implies M = {A-delta Bover 1-delta}$$
                  $$QC = delta QB implies Q = {C-delta Bover 1-delta}$$
                  $$NA = delta ND implies N = {A-delta Dover 1-delta}$$



                  thus $$MN ={A-delta Dover 1-delta}-{A-delta Bover 1-delta} = {deltaover 1-delta}DB$$ $$ = {deltaover 1-delta}varepsilon cdot AC=i{ACover 1-delta} = iMQ$$



                  and we are done.







                  share|cite|improve this answer














                  share|cite|improve this answer



                  share|cite|improve this answer








                  edited Jan 14 at 17:44

























                  answered Jan 14 at 16:32









                  Maria MazurMaria Mazur

                  49.9k1361125




                  49.9k1361125






























                      draft saved

                      draft discarded




















































                      Thanks for contributing an answer to Mathematics Stack Exchange!


                      • Please be sure to answer the question. Provide details and share your research!

                      But avoid



                      • Asking for help, clarification, or responding to other answers.

                      • Making statements based on opinion; back them up with references or personal experience.


                      Use MathJax to format equations. MathJax reference.


                      To learn more, see our tips on writing great answers.




                      draft saved


                      draft discarded














                      StackExchange.ready(
                      function () {
                      StackExchange.openid.initPostLogin('.new-post-login', 'https%3a%2f%2fmath.stackexchange.com%2fquestions%2f3069153%2fshow-that-mnpq-is-a-square%23new-answer', 'question_page');
                      }
                      );

                      Post as a guest















                      Required, but never shown





















































                      Required, but never shown














                      Required, but never shown












                      Required, but never shown







                      Required, but never shown

































                      Required, but never shown














                      Required, but never shown












                      Required, but never shown







                      Required, but never shown







                      Popular posts from this blog

                      Cabo Verde

                      Gyllenstierna

                      Albrecht Dürer